3
$\begingroup$

Hilbert introduced his famous matrix when he studied the following problem. How small can the integral $$\int_{a}^b|p(x)|^2dx $$ become for a non-zero polynomial $p$ with integer coefficients? He answered this question by showing that the discriminant of the associated quadratic form on the space of polynomials of degree at most $n$ is of the from $c_n\left(\frac{b-a}4\right)^{n^2}$ for $c_n$ tending to 1. He then applied Minkowski's theorem to deduce that, if $(b-a) < 4$, then indeed we can make the $L^2$ norm on the interval $[a,b]$ as small as we like. My question is whether the converse is true. So, if $b-a > 4$ then the $L^2$ norm is bounded from below by a positive constant. My second question, is whether there are generalizations of his result to integrals over sets other than intervals.
The article I am referring to is: ``Ein Beitrag zur Theorie des Legendre'schen Polynoms." Thanks!

$\endgroup$

1 Answer 1

3
$\begingroup$

A stronger result is true: if $b-a>4$, then the infimum of $\int |p|^2$ is positive when taken over all monic polynomials $p(x)=x^n+c_{n-1}x^{n-1}+\ldots +c_0$.

This follows from facts about orthogonal polynomials and their associated Jacobi matrices. Here we are interested in the (spectral) measure $d\rho(x)=\chi_{(a,b)}(x)\, dx$. (To make sure that the standard framework from below applies, I should actually work with the normalized version $\rho/(b-a)$, but of course this does not affect the question under consideration here.)

Consider the associated orthogonal polynomials $p_n$. These satisfy a three term (Jacobi) recurrence relation $$ a_n p_{n+1}(x) + a_{n-1}p_{n-1}(x) + b_n p_n(x) = xp_n(x) . $$ If we start with the initial values $p_0=0$, $a_0p_1=1$, then these are normalized in the sense that $\int |p_n|^2\, d\rho =1$. (This is not immediately obvious but follows from the standard machinery because $u\mapsto \sum u_n p_n(x)$ is a unitary map from $\ell^2$ to $L^2(\rho)$ and $p_n$ is the image of $e_n\in\ell^2$ and $\|e_n\|=1$.)

We can now expand a general $p$ as $p=\sum_{n=1}^N c_n p_n$. By orthogonality, $\|p\|^2_{L^2(\rho)}=\sum |c_n|^2$. Moreover, the recursion shows that the leading coefficient of $p_n$ is $1/(a_1a_2\cdots a_{n-1})$. So if $p$ is monic, then $\|p\|\ge |c_N|\ge a_1\cdots a_{N-1}$.

So it now suffices to show that $\liminf_{n\to\infty} (a_1\cdots a_n)>0$. This looks plausible because at least in friendly situations, $A=\lim (a_1 \cdots a_n)^{1/n}$, if it exists, is the logarithmic capacity of the spectrum $E$ of $\rho$.

Theorem 1.4 of my paper here establishes the stronger result that $\liminf a_n\ge (b-a)/4$ in a different way. It also gives references on how to use potential theoretic tools to obtain the weaker (but more general) version.

Finally, for an intuitive explanation of why $b-a=4$ is the critical value, recall that the capacity of an interval $[a,b]$ equals one when $b-a=4$.

$\endgroup$

You must log in to answer this question.

Start asking to get answers

Find the answer to your question by asking.

Ask question

Explore related questions

See similar questions with these tags.